Line 1: Line 1:
 +
Proof by contradiction.
  
Assume that there are only a finite number of prime number <math>p_1,p_2,p_3,......p_n</math>
 
.  Then by using the fact from exercise 18 (Let <math>p_1,p_2,p_3,....,p_n </math> be primes.  Then <math>p_1p_2.....p_n +1 </math> is divisible by none of these primes), <math>p_1p_2p_3....p_n +1</math>  is not divisible by any prime.)  This means <math>p_1p_2...p_n +1 </math> (which is larger than our initial conditions) is itself prime.  This contradicts the assumption that <math>p_1,p_2,...p_n </math> is the list of all primes.
 
  
~Angela
+
Assume that there are only a finite number of prime number <math>p_1,p_2,......,p_n</math>
 +
.  Then by using the fact from exercise 18 (Let <math>p_1,p_2,....,p_n </math> be primes.  Then <math>p_1p_2.....p_n +1 </math> is divisible by none of these primes), <math>p_1p_2p_3....p_n +1</math>  is not divisible by any prime.)  This means <math>p_1p_2...p_n +1 </math> (which is larger than our initial conditions) is itself prime.  This contradicts the assumption that <math>p_1,p_2,...p_n </math> is the list of all primes.
 +
 
 +
--Angela [[User:Akcooper|Akcooper]] 20:09, 6 September 2008 (UTC)

Revision as of 16:09, 6 September 2008

Proof by contradiction.


Assume that there are only a finite number of prime number $ p_1,p_2,......,p_n $ . Then by using the fact from exercise 18 (Let $ p_1,p_2,....,p_n $ be primes. Then $ p_1p_2.....p_n +1 $ is divisible by none of these primes), $ p_1p_2p_3....p_n +1 $ is not divisible by any prime.) This means $ p_1p_2...p_n +1 $ (which is larger than our initial conditions) is itself prime. This contradicts the assumption that $ p_1,p_2,...p_n $ is the list of all primes.

--Angela Akcooper 20:09, 6 September 2008 (UTC)

Alumni Liaison

To all math majors: "Mathematics is a wonderfully rich subject."

Dr. Paul Garrett